RA6109 Exam III: Trauma

Pataasin ang iyong marka sa homework at exams ngayon gamit ang Quizwiz!

50% *CT:* more sensitive than radiography

*Plain film* may miss up to _______ of rib fractures even with dedicated oblique rib projections.

1.*Floating rib* 2.*Flail chest*

1. What term is used to describe a rib that is fracture twice (free fracture fragment)? 2. When three or more contiguous floating ribs are present this is called?

Subcoracoid Subglenoid Subclavicular Intrathoracic (very rare)

Anterior dislocations can be typically divided into...?

*Child Protective Services*

Any suspicion of abuse must be reported to ______________ _______________ ______________.

Increasing pain in the heel with exercise or activity Tenderness to palpation over outer aspects of the heel Pain may decrease or disappear with rest *Management:* -Rest and activity modification is sufficient -Stress fracture if mismanaged, or if stresses will continue may progress to a complete fracture.

What are the clinical features of a stress fracture? Management?

Early/secondary advanced DJD *Treatment:* -Depends on the severity and complications. -Surgical care may be needed.

What are the complications of a Osteochondral injury (Talar body fracture)? What is the treatment?

Compartment syndrome Vascular injury Nerve injury *Treatment:* In most cases, surgical

What are the complications of a Supracondylar and Condyalr distal femur fracture?

1. Epidural hematoma 2. Subdural hematoma 3. Subarachnoid hemorrhage 4. Pneumocephalus 5. Meningitis 6. Leptomeningeal cysts (growing fracture)

What are the complications of skull fractures?

1. Nasal bones 2. Orbital fractures 3. Tripod fracture 4. LeFort fracture 5. Mandibular fractures

What are the different type of fractures of the facial skeleton?

Type I: Wedge (low force) Type II: Wedge with depression Type III: Depression with no wedge

What are the first 3 Shatzker types of Tibial Plateau Fracture?

1. Direct blow (or fall directly on the elbow) 2. FOOSH with elbow flexed 3. Avulsion by the Triceps muscle 4. Stress fracture, e.g. throwing athletes and gymnasts *Important to remember that the ulnar nerve is closely related to the medial aspect of the Olecranon and can be occasionally damaged*

What are the four mechanisms of an Olecranon fracture?

Type IV: Similar to type 1 but involves the medial plateau as a result of a greater force Type V: both plateaus Type VI: Both plateaus and a part of proximal metaphysis or diaphysis

What are the last 3 types of Shatzker fractures (Tibial Plateau Fracture)?

Iliac Crest -Abdominal muscles ASIS -Sartorius AIIS -Rectus femoris Greater Trochanter -Gluteus medius & minimus Lesser Trochanter -Iliopsoas Symphysis -Adductor Ischial tuberosity -Hamstring

What are the locations and associated muscles of Stable Pelvic fractures (avulsion)?

*Immediate:* -Neurological and vascular injury, acute compartment syndrome and renal failure, pulmonary fat embolism, gas gangrene *Intermediate:* -Osteomyelitis, sepsis, complex regional pain syndrome formerly known as RSDS, non-union/malunion *Late:* -Ischemic necrosis (AVN), Secondary osteoarthritis *Gun shot wound (GSW):* -Associated fractures may sometimes cause lead toxicity: if the bullet was lodged in the synovial or serous cavity (e.g. peritoneal cavity etc.), hyaluronic acid will chemically remove lead and cause its reabsorption into general circulation potentially causing lead toxicity.

What are the major complications of fractures?

4th-10th ribs

What are the most commonly fractured ribs?

Calcaneus (M/C) -2% of all fractures -60% of all tarsal fractures Talus

What are the most frequently fractured bones of the foot?

In adults: C1-C2 distance on lateral view is ~2.5-mm and should not increase on flexion In children: it may be up to 5-mm and may change by 1-2-mm in flexion

What are the normal Atlanta-dental interspace or (ADI) measurements for adults and children?

Bennett fracture dislocation Rolando fracture Gamekeeper' s thumb (not always includes a fracture)

What are the other names for a fracture at the base of the 1st metacarpal of the thumb?

Compression wedge and burst fractures (discussed already) Other significant injury known as *Seat-belt or Chance fracture* *Chance fracture-* flexion-distraction type injury of the spine that extends through to involve *all three spinal columns* *Unstable injury* and have a high association with intra-abdominal trauma (esp. pancreatic and duodenal injury, renal vascular Pericles injury may also occur)

What are the other types of thoracic, thoracolumbar, and lumbar fractures?

When injury and intra-osseous edema have occurred. Osteoclasts will become activated by the inflammatory mediators released from injured cells. As a result, more significant bone resorption will occur and what was initially undetected on the x-ray as bone bruise may progress to fracture several days later. Best example of an occult fracture is injuries to carpal navicular or scaphoid bone

What is the definition of an Occult fracture?

Avulsed medial epicondylar fragment with intra-articular interposition

What is the diagnosis?

Avulsion in Gamekeepers thumb

What is the diagnosis?

Avulsion with displacement of the olecranon -Triceps muscle

What is the diagnosis?

Barton Fracture -Dorsal line and intra-articular wrist extension

What is the diagnosis?

Bennett fracture-dislocation -Note *2-piece fracture* that should help DDx it from Rolando or comminuted Bennett.

What is the diagnosis?

Boxer Fracture -Mild impaction with volar angulation at distal metaphysis

What is the diagnosis?

Chauffeur fracture

What is the diagnosis?

Colles fracture of distal radius

What is the diagnosis?

Comminuted Olecranon fracture (direct blow)

What is the diagnosis?

Comminuted intercondylar fracture

What is the diagnosis?

Displaced radial head fracture

What is the diagnosis?

Dorsal triquetral fracture

What is the diagnosis?

Fibrodysplasia Ossificans Progressive (FOP)

What is the diagnosis?

Galeazzi fracture-dislocation

What is the diagnosis?

Galeazzi fracture-dislocation with volar radial displacement (Type II)

What is the diagnosis?

Gamekeeper thumb (avulsion by the ulnar collateral ligament)

What is the diagnosis?

Garden Type II Proximal Femoral Neck Fracture -Complete -Non-displaced

What is the diagnosis?

Garden Type IV Proximal Femoral Neck Fracture -Complete ->50% displaced -May resemble a pathological fracture

What is the diagnosis?

Grade III ACJ injury

What is the diagnosis?

Grade III ACJ injury-separation Note disruption in the undersurface of distal clavicle-acromion

What is the diagnosis?

Humeral shaft fracture -middle third

What is the diagnosis?

Impaction and dorsal angulation of distal radial fracture (Colles fracture) *Ulnar styloid is involved = Moor fracture*

What is the diagnosis?

Inferior GHJ dislocation (Luxatio Erecta)

What is the diagnosis?

Jersey Finger

What is the diagnosis?

Lunate Dislocation -Note classic lunate dislocation sign referred to as "*pie sign*" when lunate overlaps the capitate with characteristic triangular appearance

What is the diagnosis?

Mallet finger avulsion

What is the diagnosis?

Monteggia fracture-dislocation

What is the diagnosis?

Nightstick Fracture

What is the diagnosis?

Nightstick fracture

What is the diagnosis?

PIP joint dislocation

What is the diagnosis?

Pisiform fracture

What is the diagnosis?

Posterior elbow dislocation with fracture of he coronoid process of the ulna

What is the diagnosis?

Proximal humeral Fracture Greater Tuberosity Fracture Neer one-part fracture

What is the diagnosis?

Proximal humeral fracture Displaced Surgical neck fracture Neer two-part fracture

What is the diagnosis?

Proximal humeral fracture Neer One-part fracture -Less than 45 degree angulation and 1-cm displacement

What is the diagnosis?

Reverse Barton Fracture

What is the diagnosis?

SLAC wrist and DISI -Due to AVN -Note advanced OA and multiple fragmentation and loose bodies

What is the diagnosis?

Scapholunate dissociation -Due to ligament rapture and instability leading to *signet ring sing d/t rotatory subluxation of the scaphoid and *Terry Thomas sign* as widened scapholuante space >3-mm

What is the diagnosis?

Scapula fracture

What is the diagnosis?

Smith fracture (or reversed Colles fracture) -Note extra-articular distal radius fracture with volar angulation and comminution

What is the diagnosis?

Supracondylar fracture -Posteriorly displaced -Fall on the flexed elbow

What is the diagnosis?

Transcondylar fracture with displacement

What is the diagnosis?

Traumatic diastasis of symphysis pubis

What is the diagnosis?

Volar or reverse Barton fracture (more accurately known as Smith type 2 fracture)

What is the diagnosis?

Myositis Ossificans (MO) Zonal phenomenon

What is the diagnosis? What phenomenon is present that will help to DDx from other aggressive lesions?

Myositis Ossificans (MO) Cleft-string sign

What is the diagnosis? What sign is present?

Shenton line

What is the dotted yellow line indicating?

Dupuytren Fracture

What is the eponym for a bimalleoular ankle fracture?

Colles fracture -FOOSH -Very common extra-articular fracture of the distal radius

What is the eponym for distal radial fracture?

Smith fractures (aka Goyrand fracture)

What is the fracture called of the distal radius with associated *palmar (volar)* angulation of the distal fracture fragment?

Surrounding: -Muscles -Ligaments *As a result, fracture may always involve significant soft tissue damage*

What is the initial energy of trauma being absorbed by?

Vertically oriented zone of increased density on lateral radiograph that is perpendicular to the trabecular lines. -Appears as Sclerosis *MRI:* -Is considered most sensitive and specific imaging of stress fractures of the lower extremity

What is the key feature of a Calcaneal stress fracture?

1.5 to 3 cm (2-cm) distal to the proximal tuberosity at the *metadiaphyseal junction* without distal or intra-articular extension.

What is the key feature of a jones fracture (Dance Fx)?

Inferior Shoulder Dislocation (aka *Luxatio erecta*)

What is the least common should dislocation?

L3 burst fracture with retropulsion of posterior body fragments into canal potentially creating neurological damage to cord or nerve roots

What is the level and name of the fracture that is present?

Os trigonum

What is the main DDX of Posteriolateral talar process fracture?

Traumatic disruption of bone and periosteum causes significant hemorrhage that initiates fracture healing.

What is the meaning of Fracture healing?

Occur as a result of significant adduction force to the forefoot with the ankle in plantar flexion.

What is the mechanism if injury in a Jones fracture?

Usually post direct trauma to he patella or sudden forceful con action of the quadriceps muscles *Clinical presentation:* -Marked swelling and pain over the patella with point tenderness and marked reduction in extension strength -Large joint effusion or hemarthrosis

What is the mechanism of a Patella fracture?

*Uncommon* Associated with sports and jumping Avulsion occurs with violent quadriceps muscle contraction or passive flexion against contracted quadriceps muscles Young children with ongoing Osgood-Schlatter disease *Treatment:* -conservative with rest and activity modification

What is the mechanism of a Tibial tuberosity avulsion fracture?

Occurs when the foot is dorsiflexed and inverted, as can happen with snowboarding. *Treatment:* -Non weight bearing cast for 6 weeks. -Unless Displaced or involving significant portion of the talar side of the posterior facet then surgery with ORIF.

What is the mechanism of a snowboarder fracture? Treatment?

Direct blow or following a fall onto the shoulder with an adducted arm. This pushes the acromion forcibly Inferiorly and medially with respect to the clavicle.

What is the mechanism of action for an Acromioclavicular joint injury?

Caused by hyperabduction/inferior push of the arm that forces the humeral head against the acromion as a result of abduction and Inferiorly applied force.

What is the mechanism of action for an Inferior Shoulder Dislocation?

Humeral head is forced posteriorly in *internal rotation* while the arm is being *abducted*

What is the mechanism of injury for a Posterior Shoulder Dislocation?

*CT scanning*

What is the modality of choice when viewing the a sternal fracture?

Surgical neck fracture

What is the most common Neer two-part fracture?

Radial nerve in the spiral groove

What is the most common associated injury during Humeral shaft fractures is a damage to..?

*Convulsive disorder (epilepsy)* Electrocution is a known classic but rare cause of posterior shoulder dislocation

What is the most common cause of Posterior Shoulder Dislocation?

Motor Vehicle Accidents (MVA)

What is the most common cause of Sternal fractures?

Excessive inversion 1. Medial malleolus is shorter than the lateral malleolus, allowing the talus to invert more than evert. 2. The deltoid ligament stabilizing the medial aspect of the ankle joint offers stronger support than the thinner lateral ligaments. As a result, the ankle is more stable and resistant to eversion injury.

What is the most common cause of ankle injuries? Why?

*Pseudo-Jones Fracture* -Over 90% of fractures of the base of the 5th metatarsal.

What is the most common foot avulsion injuries?

Bilateral fracture of the posterior arch of C1 Should not be confused with Jefferson C1 fracture Mechanism: Extension/compression of the C1 arch by occiput *Image represents:* C1 posterior arch fracture (not Jefferson)

What is the most common fracture of the Atlas (>50%)

Middle third: 60% (commonest) Proximal third: 30% Distal Third: 10%

What is the most common location of Humeral shaft fractures?

Cutaneous injury *Fractures are noted in 30% (range 11-55%)

What is the most common maltreatment of children?

FOOSH

What is the most common mechanism of a Monteggia fracture-dislocations?

Axial loading -Fall from a height (most common)

What is the most common mechanism of proximal tibial fractures?

Anterior GH dislocation -Forced abduction, external rotation and extension

What is the most common shoulder dislocation?

2nd and 3rd MT fractures *March Fracture*

What is the most common stress fractures of the metatarsals?

Horizontal (image) or Cleavage tear Best evaluated with MRI *Posterior horn of medial meniscus is torn much more frequently* due to its fixed attachment to corresponding tibial plateau.

What is the most common type of Meniscal tears?

*Lisfranc injuries* or *Lisfranc fracture-dislocations Mechanism: -Varies = direct crush injury or an indirect load onto a plantar flexed foot.

What is the most common type of dislocation involving the foot?

Scaphoid fractures (70-80%) -FOOSH *-Scaphoid fat pad sign*

What is the most commonly fractured carpal bone?

Anterior Collateral Ligament (ACL) Involves rapid starting, stopping, and pivoting -Soccer, Basketball, Tennis *MRI is required for diagnosis and management* Note: ACL tear in image

What is the most commonly torn ligament of the knee?

Lunate

What is the most frequently dislocated carpal bone?

5th metacarpal (25% of all metacarpal fractures)

What is the most frequently fractured metacarpal?

*Widening of the AC joint* *Normally undersurface of the acromion should be leveled with the undersurface of the clavicle*

What is the most significant feature of a ACJ injury?

Pulled elbow (aka *Nursemaid's elbow*)

What is the name of the fracture that is a subluxation of the radial head out of the annular ligament, which usually spontaneously or easily reduces and rarely demonstrates abnormal radiographic features.

Nightstick fractures (aka Parry or Tap fx)

What is the name of the isolated fractures of the ulna, typically in the mid-diaphysis, usually resulting from a direct blow. It is a characteristic defensive fracture when the patient tries to ward off an overhead blow from an assailant.

Jefferson Fracture APOM view: overhanging C1 masses, *if >6-mm combined, suspect transverse ligament damage and marked instability*

What is the name of this fracture?

Jefferson Fracture C1-C2 ADI widening and fracture line through posterior arch of C1

What is the name of this fracture? *hint: ADI*

Jefferson Fracture Patient does not show neurological signs

What is the name of this fracture? Does the patient sho neurological signs?

Adults: -No more than 22-mm at C6-C7 *At C6 or C7 soft tissue should measure less than the width of the adjacent vertebral body* -No more than 7-mm at C2 *alternatively soft tissue thickness at C2 should measure less than 50% of vertebral body width* Abnormal widening of prevertebral soft tissues indicates pathological process -Post traumatic hematoma or a mass of different origin i.e. infection vs. neoplasm in non-traumatic cases

What is the normal retropharyngeal & retrotracheal soft tissue measurements in adults?

Positive Posterior fat pad sign

What is the orange arrow indicating?

Trough sign Posterior Shoulder Dislocation (PSD)

What is the orange arrow indicating? What is the diagnosis?

*Bumper or Fender fracture* But only 25% of tibial plateau fractures result from impact with automobile bumpers

What is the original term for a Tibial plateau fracture?

Insertion of *Peroneus brevis* and forceful inversion of the foot in plantar flexion. -It is also relatively common among tennis players and occasionally being referred to as a *"Tennis feature"*

What is the pathology of a Pseudo-Jones Fracture?

Sternal fracture

What is the primary reason the patient is being X-rayed?

Malunion of distal tibial and fibula Hypertrophic exuberant calus

What is the problem of this disturbed fracture healing?

10% of people with a proximal femoral neck fracture die within one month Approx. 33% die within 12 months

What is the prognosis for a more complex hip fracture in the elderly?

1 per 1000 in Western Countries

What is the rate of incidence for a Proximal Femoral fracture?

Herbert screw

What is the screw called that is used for a non-union scaphoid injury?

Fracture of Posterior talar process (solid white arrow) Marked effusion of pretibial fat pad (hollow white arrow) Kagar's fat pad (black arrow)

What is the solid white arrow indicating? Hollow white arrow? Black arrow?

Un-displaced radial Head fracture (Mason type 1)

What is the specific diagnosis?

*Treatment:* -Conservatively -If large or very displaced fragments with intra-articular extension then operative fixation may be indicated.

What is the treatment and prognosis of a Pseudo-Jones Fracture (Tennis fracture)?

If stable conservative care can be considered If ligaments are disrupted, neurosurgical treatment with fixation stabilization of C1 and C2

What is the treatment for a Jefferson fracture?

Contrast to avulsion fractures *Jones fractures are prone to non-union (with rates as high as 30-50%)* and almost always take longer than two months to heal. As displacement of the fracture can be increased with persistent weight bearing, immobilization is important as part of the initial therapy, with a non-weight bearing cast for 6-8 weeks.

What is the treatment for a Jones fracture?

If no complications present a *Kocher's maneuver* can be performed by traction and external rotation and then adduction and internal rotation.

What is the treatment for shoulder dislocations?

Avulsion Fracture Inferior patella pole

What is the type of fracture?

Surgical fixation

What is the typical treatment for a femoral shaft fracture?

Volar direction

What is the typically angulation of a Boxer fracture?

Powerful musculature of the thigh

What is the usual cause of frequent displacement of the femur?

Transverse sacral fracture CT with coronal reconstruction

What is the white arrow indicating?

Os trigonum -10 year old patient -Ankle pain -Ossicle differentiated by well croticated margin and no other evidence of trauma

What is the white arrow pointing at?

Fracture of the coronoid process of the ulna

What is uncommon and often occur in association with *posterior elbow dislocation?*

PIP

What joint is most commonly involved in Interphalangeal joint dislocations?

ACL Tear MCL Tear Medial Meniscus tear (bucket handle tear) Due to pivot shift and strong valgus force

What ligaments does the O'Donogheu's unhappy triad include? What is it typically due to?

B/L parasymphyseal/mental mandibular fracture

What mandibular fracture is present?

An osteochondral fragment *(Kocher fracture)* may be sheared off the articular surface of he capitellum

What may also develop with a condylar distal humeral fracture?

Radial nerve injury

What may be a complication in this spiral humeral shaft fracture?

5th MT fractures and non-unions -Injuries can be induced by dropping heavy objects or inversion and eversion injuries in dorsiflexion or plantar flexion.

What metatarsal is fractured most frequently?

*Sciatic Nerve* Can be potentially damaged and some patients may present clinically as unable to ambulated or even move their extremity with regional paresthesia In addition to severe pin and often other associated trauma elsewhere

What nerve is at risk during a posterior hip dislocation?

Malgaigne fracture -1/3 of all pelvic fractures

What pelvic fracture is the most common?

*Only 2-4%*

What percentage of Posterior shoulder dislocation account for shoulder dislocations?

70%

What percentage of Proximal humeral fractures occur in women with osteoporosis?

90%

What percentage of proximal humeral fractures occur at home due to falls (e.g. FOOSH)?

3-5% of all fractures Males in their 3rd decade and patients >65+

What percentage to Humeral Shaft fractures account for?

The *pronator quadratus sign* Indirect sign of distal forearm trauma Relies on displacement of the fat pad that lies superficial to the pronator quadratus muscle On lateral wrist radiographs, the pronator fat pad normally appears as a thin radiolucent stripe, with its base attached to the palmar surface of the radius dismally and a thin line parallel to the distal radius it is seen normally in 90% of subjects *Other causes apart from trauma may include:* -Hematoma -Inflammation -Infection and others

What radiographic sign is present?

*PA ulnar deviation view of the wrist*

What radiographic view is required for proper evaluation?

Mach line (effect) Not to be confused with Dens fracture

What radiological effect is present on this X-ray?

Sternal fracture *Sternal foramen (or perforated sternum)*

What should this not be confused with? What is this DDx?

Left: -Epidural hematoma -Middle meningeal artery Right: -Subdural hematoma -Bridging Veins

What side is an Epidural Hematoma and Subdural hematoma? What arterial is involved?

Hawkins sign -Reduced risk of AVN

What sign are the arrows pointing to?

FBI Sign Indication of intra-articular/intra-capsular fracture about the knee, typically tibial plateau fractures

What sign is present in this image?

Sacral insufficiency fracture *(Honda sign)*

What sign is present on this radionuclide bone scan?

*"Black eye brow sign"* of orbital emphysema d/t *Blow out fracture*

What sign is present?

Posterior fat pad sing (orange line) Anterior fat pad sign (sail sign) Evidence of anterior humeral line

What signs are present?

Midshaft clavicular fracture

What type and region of fracture?

Stellate Patella fracture Usually direct impact and burst Quadriceps forces may cause separation of fragments

What type of Patella fracture is present?

Salter-Harris type 1 Fracture through growth plate itself often unrecognized because of minimal displacement

What type of Salter-Harris Fracture?

Salter-Harris type 2 Most common (>75%) Fracture through physis and a part of metaphysis forming a Thurston-Holland fragment. Good Healing prospects.

What type of Salter-Harris Fracture?

Salter-Harris type 3 Through the epiphysis with intra-articular extension

What type of Salter-Harris fracture?

Salter-Harris type 4 Fracture traverses metaphysis, physis, and into epiphysis Unfavorable prognosis

What type of Salter-Harris fracture?

Salter-Harris type 5 Crush injury to growth plate, often unrecognized or confused with type 1 but essentially damages physical blood supply. Unfavorable prognosis of premature plate closure.

What type of Salter-Harris fracture?

Salter-Harris Type II Thurston-Holland fragment

What type of Salter-Harris fracture? What is the black line indicating?

Varus angulation Bayonet deformity Butterfly fragments

What type of angulation deformity is present?

Valgus Deformity with Posterior Angulation

What type of deformity? What type of angulation?

Posterior Hip Dislocation

What type of dislocation is depicted in the X-ray?

*Subcoracoid* Anterior Shoulder Dislocation (Most common)

What type of dislocation is present?

Supracondylar fracture in osteoporotic elderly person with less severe split and no evidence of major displacement Low force or even trivial trauma leading to fracture in osteoporotic patients is a key mechanism of injury

What type of distal femur fracture is present? Mechanism of fracture (young or old)?

Supracondylar high force comminuted fracture in younger patient Demonstrating more significant split and separation of fracture fragments due to major violence Surgical care is required

What type of distal femur fracture is present? Mechanism of fracture (young or old)?

Nasal bone fracture *M/C* of all facial bone fracture (>45%) and can be missed d/t large regional swelling Result of *direct impact* by fist, dash board, steering wheel etc. Conventional radiographs ~80% sensitivity CT scan 100% sensitivity -E.g blow-out fractures

What type of facial fracture is present?

Orbital blow-out fracture Non-contrast CT

What type of facial fracture is present?

Orbital blow-out fracture A result of sudden impact to the globe Most cases will result in infra-orbital fracture that will force orbital fat and sometimes inferior rectus (IR) muscle into the maxillary sinus (occasionally IR may be trapped-*"trapped-door" fracture*) Orbital emphysema may develop 50% of cases may involve medial wall (lamina papyrecea) fracture *Best imaging is CT* of the facial Skelton and sinus with coronal reconstruction CT helps to exclude complications like optic nerve injury, trapped IR muscle etc. *Water's skull view* may show opacification of the maxillary sinus *"Tear drop" sign*

What type of facial fracture is present? What radiographic view is this? What sign is the long arrow pointing to?

LeFort fracture From L to R LeFort I: -horizontal maxillary fracture producing *separation of the upper teeth* from the face, passing through the alveolar ridge, lateral nose and inferior wall of maxillary sinus LeFort II: -Pyramidal-shaped fracture with the upper teeth at the base and the nasal-frontal suture at the apex, passing through posterior alveolar ridge, lateral maxillary sinus wall, inferior orbit and nasal bones LeFort III: -*Cranial-facial spearation*, fracture passing through nasal-frontal suture, maxilla-frontal suture, orbital wall and zygomatic arch

What type of facial fractures?

Garden Type II Proximal Femoral Neck Fracture -Nondisplaced -Complete

What type of fracture are the red arrows indicating?

Maisonneuve fracture

What type of fracture does this proximal oblique fibula fracture indicate?

Stubbed toe fracture of 4th proximal phalanx (Bedroom fracture)

What type of fracture is in the image? Location?

Chance or seat belt fracture Radiographically anterior body narrowing and fracture through posterior elements

What type of fracture is present ?

5th MT avulsion non-displaced (Pseudo-Jones Fracture) Location and fracture position is the *key DDX* from Jones.

What type of fracture is present in the image?

Osteoporotic sacral insufficiency fracture

What type of fracture is present in this post menopausal woman?

*Metaphyseal fracture* (aka Bucket handle fracture or corner fracture)

What type of fracture is present in up to *39-50%* of abused infants <18 months?

*Clay-shoveler fracture* Fractures of the SP of a lower cervical vertebra (usually C7), and are usually an avulsion-pull fracture Clinical Presentation: Often unrecognized at the time and only found incidentally years later when the cervical spine is imaged for other reasons. Causes: Motor vehicle accidents, sudden muscle contraction, direct blows to the SP Radiographically: Seen on lateral views as an oblique lucency through the SP, usually of C7. There is usually displacement of SP It was originally described by Australia due to stickiness of clay and maximum muscular contraction effort required by the posterior back muscles causing an avulsion of the SP

What type of fracture is present?

*Greenstick fracture* Incomplete fractures of long bones Usually seen in young children, typically less than 10 years old Typically at mid shaft or meta-diaphysis in the forearm and/or lower leg Most occur during bending and compressive forces with fracture of the convex side and are usually unicortical (i.e affecting a single cortex) The reason for incomplete fractures is believed to be the result of an immature bone being different to mature bone with more elastic collagenous cortex and loosely attached periosteum Green stick fractures are considered *less frequent* then a Buckle or Torus fractures *Bowing or Plastic deformity of the involved bone be also co-exist

What type of fracture is present?

2nd MT stress fracture evolution -Early diagnosis can be helped by MRI *Radiography:* -Typical changes with solid periosteal response with or without radiolucent line. *-This is a classic March fracture in the 2nd MT (commonest)*

What type of fracture is present?

5th MT avulsion, mildly displaced Located more proximal at the tuberosity and styloid of the 5th MT DDX from Jones.

What type of fracture is present?

Tripod fracture (PA Water's view) *2nd m/c facial fracture after nasal fracture (40% of mid face fracture)* M/c mechanism is a blow to malar region of the face Complications: -Trismus -Trapped temporalis msucle -loss of sensory distribution along maxillary (V2) nerve distribution *Best Imaging modality* CT scan

What facial fracture is present?

Kagar's fat pad Should prompt active search for fracture.

What fat pad in this image is marked by opacification? What does the opacity of this fat pad indicate?

Extension teardrop fracture of the vertebral body of C2 Multiple CT slices in bone window revealing anterior inferior corner avulsion

What fracture is present and at what level?

Flexion teardrop fracture and associated cord damage

What fracture is present on this MRI?

B/L facet dislocation at C6 with 50% anterior body displacement Note the facet degeneration may predispose to dislocations as seen in this case

What fracture is present?

C1 Posterior Arch fracture (not Jefferson)

What fracture is present?

Chance fracture or Seat belt fracture Sagittarius T2 MRI

What fracture is present?

Coccygeal fracture Infrequent and rarely diagnosed on pelvic radiographs Coccyx views may be more helpful Most coccyx fractures result from falls on the buttock and direct impact Clinically localized pain and tenderness are present When diagnosed the treatment is conservative The need of X-ray examination in suspected coccygeal fractures is questioned by many since conservative care is usually given

What fracture is present?

Hyperextension of the C/S may cause *avulsion* of the inferior-anterior corner of the vertebral body May be stable in flexion, but unstable in extension Association with Hangman's fracture may occur, especially in older patients due to *spondylosis* of the C/S and C2 becoming more vulnerable to excessive stresses In the presented image: Associated extension teardrop fracture and Hangman's hyperextension injury. this is an avulsion of C2 or C3 body by ALL

What is an Extension Teardrop Fracture?

Break and/or *disruption in 2-places or more

What is an Unstable pelvic fracture?

Fragment of bone being detached by the tension from muscles or ligaments.

What is an avulsion fracture?

B/L facet dislocation due to tear of capsular facet ligaments. Unstable injury Note 50% anterolisthesis of C5 on C6 and overriding facets

What type of fracture is present?

Burst Fracture Note *interpediculate widening on AP view* and posterior body displacement on the lateral

What type of fracture is present?

*T*ransverse fractures of the proximal tibial metaphysis -That occur in children (typically) while jumping on a trampoline (or bouncy castle) -often seen in *children 2 to 5 years of age*

What is a Trampoline fracture?

Involves *avulsion* of the tip May be unstable Less common

What is a Type I Odontoid process fracture?

Through the base of the odontoid process and may involve crucial events ligament Can be missed (>23% missed cervical fractures) *Most unstable* *Most common*

What is a Type II Odontoid process fracture?

Through the base into the body and lateral masses. Can be stable and carries best healing potential due to greater fracture surface and vascularization/

What is a Type III Odontoid process fracture?

Proximal humeral fractures

What is a common UE fracture especially in 65+ years?

Typically in the calvaria (cranial vault) and occasionally in Tibial plateau.

What is a depression fracture?

A *single break* -about 2/3 of pelvic fractures -Pelvic avulsion fractures, iliac wing fracture, sacrum fractures, and ischiopubic ramus fracture

What is a stable pelvic fracture?

Repeated stress applied to normal bone leading to bone marrow hyperemia and bone resorption (e.g. March fracture of 2nd or 3 digit metatarsal bone, Tibial stress fractures in runners etc.) What is a stress (fatigue) fracture?

What is a stress (fatigue) fracture?

Results in telescoping of osseous trabeculae. No typical radiolucent line is seen on radiographs and instead a zone of sclerosis or condensation by be present.

What is the definition of Impaction (compression) fracture?

Bone weakened by pathology such as neoplasms, infection, congenital defect of collagen etc.

What is the definition of Pathologic fracture?

Partial loss of articular alignment

What is the definition of Subluxation?

Complete loss of articular contact/alignment with resultant injury to periarticular restraints

What is the definition of a Dislocation?

Complete or incomplete disruption in continuity and structure of the bone and/or cartilage.

What is the definition of a Fracture?

Result of head injury that must be evaluated with the highest level of clinical suspicion

What is the definition of a Skull fracture?

Transverse, Oblique, and Spiral

What is the definition of a complete fracture?

Jones Fracture *Non-union Fracture8 When fracture healing does not occur within *6-9 months* 5th Metatarsal (Jones) fracture Occurs due to lack of good immobilization and impeded healing process in the bone preventing normal callus formation and healing

What type of fracture needed surgical intervention is present?

Green Stick Fracture -Distal ulna metadiaphysis Plastic Deformity -Radius

What type of fracture(s)?

Bayonet Apposition of fracture fragments, when fragments override

What type of fracture?

Chip Fracture Considered: "Chip" corner fracture

What type of fracture?

Compression (Insufficiency) Fracture Vertebral body in osteoporotic spine

What type of fracture?

Depressed Fracture *Most seen in flat bones.*

What type of fracture?

Green Stick Fracture (incomplete)

What type of fracture?

Green Stick Fracture of both bones -Radius & Ulna

What type of fracture?

Hawkins Type II displaced talar neck fracture with subtalar extension

What type of fracture?

Impaction Fracture A. Distal radius impaction fracture and foreshortening B. Impaction/compression fracture of the anterior humeral head due to posterior shoulder dislocation and compression by posterior glenoid rim so-called "trough sign" during posterior GHJ dislocation

What type of fracture?

Impaction/Depressed Fracture Tibial Plateau impaction/depressed fracture by the lateral femoral condyle. Often comminuted depending on the magnitude of forces and bone resilience

What type of fracture?

Insufficiency Fracture (arrows) Example of osteoporotic tibial insufficiency fracture when demineralized bone is unstable to adequately respond to normal mechanical stresses

What type of fracture?

March Fracture of the 3rd MT bone

What type of fracture?

Oblique Fracture

What type of fracture?

Orbital blow-out fracture (Right orbital injury) *Plain films no longer have a real role to play in the assessment of facial trauma* *"black eye brow sign"* *"tear drop" sign*

What type of fracture?

Pathologic Fracture In the bone weakened/destroyed by the pre-existing pathology (chondrosarcoma of proximal femur)

What type of fracture?

Pseudo-fracture This is a subtype of insufficiency fracture that develops in bones with insufficient osteoid. Most seen in Rickets & Osteomalacia Hyperparathyroidism, Renal osteodystrophy, Piaget Disease and other conditions that disturb normal bone physiology Radiological Key Characteristic: -Widened transverse radiolucent lines oriented at the right angle typically to the medial cortex Often referred to as Looser zones, Milkman lines or umbau zones

What type of fracture?

Spiral Fracture

What type of fracture?

Stress (fatigue) Fracture -When normal bone is exposed to excessive usually repetitive stress

What type of fracture?

Torus (Buckle) Fracture -Due to trabecular telescoping and buckling

What type of fracture?

Transverse Fracture

What type of fracture?

Pseudo-fracture Arrow is pointing to looser' s zone

What type of fracture? What is the arrow indicating?

Pseudo-fracture Milkman lines

What type of fracture? What is the key radiographic characteristic present?

Subdural hemorrhage

What type of hemorrhage in a child should be viewed with highest level of suspicion?

Chopart Injury

What type of injury is present?

"3-column injury" with posterior displaced body fragment and fanning of SP indicating severe *unstable* injury consistent with *Flexion teardrop fracture*. Note associated high signal and cord damage in relation to this fracture.

What type of injury is presented?

Left: Posterior Dislocation of the knee Right: Anterior Dislocation of the knee

What type of injury?

Traumatic Suture Diastasis

What type of injury?

Left condylar neck fracture on panorex tomographic view

What type of mandibular fracture is present?

Bisphosphanates

What type of medications in the treatment of osteoporosis may result in characteristic femoral shaft fractures?

Transverse Patella Fracture *Most Common* Exerted by sudden forceful quadriceps contraction Note large effusion and possibly haematoma

What type of patella fracture is present?

Posterior rib fractures

What type of rib fractures do not occur during CPR?

Transverse Sacral fracture and associated L4 TP fracture L4 and/or L5 TP fractures should alert for the possibility of sacral fracture

What type of sacral fracture is present and what associated fracture is present?

C6 unilateral facet dislocation and bow tie sign -Note abrupt change in facet orientation and spinolaminar line disruption -On APLC not SP alignment is lost abruptly

Where is the fracture? What type of fracture is present? What radiographic sign is present?

Waist of scaphoid: 70-80% (highest risk of AVN) Proximal pole: 20% (highest risk of non-union) Distal pole: 10%

Where on the scaphoid is it most commonly fractured?

Avascular Necrosis (AVN) Neer in 1970 -fragments are considered displaced if there is >1 cm of separation and/or >45-degrees of angulation of a fragment.

If a proximal humeral fracture is displaced, it increases the chances of what in the *humeral head?*

Odontoid fracture

If the Odontoid process leans more anteriorly, what should you suspect?

Intra-articular Proximal Femoral Neck Fracture

Image shows Advanced AVN and severe DJD. Why did this occur?

Neer Three-part fracture Proximal humeral fracture

Image shows a three part fracture across the surgical neck and one of the tuberosity. What is the classification?

Note retropulsed L2 posterior body fragments responsible for post-traumatic spinal canal narrowing and possible neurological and/or vascular complications that at L2 level may involve conus medularis

Important Information.

*Proximal pole will have no blood supply*

Important concept

More complexed toe injury with articular and nail involvement.

Important to recognize.

Displaced posterior, superior, and slightly lateral to the acetabulum and also internally roared hence the lesser trochanter is usually obscured on AP view In a well centred AP film the posteriorly dislocated femoral head will appear smaller than the contralateral hip CT scanning is required to evaluate this complex injury

In a posterior hip dislocation, the femoral head is displaced in what direction?

Motor vehicle collision (50%) Pedestrian vs. Motor Vehicle (30%) Fall from height (10%) Motorbike collisions (4%) Other e.g. sports injury, low-energy fall Pelvic insufficiency fractures are common in elderly

Most pelvic fractures result from what?

True

T/F: Position of fractures based on regions of the bone (metaphysis vs diaphysis and epiphysis) it must be stated what region of the bone is affected by the fracture.

True Gauze padding is inserted between the toes to prevent maceration, and mail beds are exposed to avoid concealing rotational deformities.

T/F: Stable toe fractures can be treated with Buddy taping.

True (80%) Anterior hip dislocation (~10%) Inferior (obturator) hip dislocation (rare) Superior (pubic/iliac) hip dislocation (N/A) Central hip dislocation (protrusio) always associated with acetabular fracture MVA and dashboard injury when hip is flexed can drive femur posterior often fracturing posterior wall of the acetabulum and potentially damaging Sciatic nerve and regional vessels

T/F: The most common hip dislocation is posterior hip dislocation.

Scapholunate dissociation

Terry Thomas sign indicates...?

*Posterior* -Larger arch -Composed of iliac wings and sacrum -Suspended by some of the strongest (sacroiliac) ligaments in the body *Anterior* -Smaller arch -Pubic and the ischium bones connected by pubic ligaments and the symphysis pubis

The anatomic pelvic ring is composed of what 2-arches?

Medial malleolus Posterior aspect of the tibial plafond (referred to as the posterior malleolus) Lateral malleolus *Considered Unstable*

What 3 bones does a Trimalleolar fracture involve?

Pelvic inlet/brim Obturator foramen

What 3-bony pelvic ring-lines are evaluated on pelvic fractures to establish continuity or disruption?

Proximal Humeral Fracture Two-part Neer fracture Surgical Neck

What Neer classification is present?

May occur due to flexion or extension or a combination of forces

What ROM can cause an Odontoid process fracture?

Weber C Ankle views -May only show widening of the ankle joint due to the distal tibiofibular syndesmosis or deltoid complex disruption. -The entire fibula needs to be imaged *Clinically:* Ankle pain tends to divert clinicians attention from a fracture more proximally.

What Weber classification is a Maisonneuve fracture considered?

Weber C -Lateral malleolus is fractured above syndesmosis.

What Weber classification would a Bimalleolar fracture receive?

Subtalar joint

What ankle joint allows for inversion and eversion?

Fractures of the pelvis and hip -Common among physically active adolescents and young adults -Sudden forceful muscular contraction causes the apophysis where the tendon attaches to avulsion *Radiographically:* -Following avulsion the avulsed bone will appear amorphous and will gradually remodel with the donor site -It may be occasionally mistaken for the aggressive lesion like *osteosarcoma* Treatment: -Managed by rest and analgesics -Surgical care ay be required in complex cases

What are Apophyseal avulsion?

*Type I:* -Non displaced fracture *Type II:* -Displaced fracture with subluxation or dislocation of the subtalar joint and a normal ankle joint *Type III:* -Displaced fracture with body of talus dislocated from both subtalar and ankle joint. AVN risk increases with increase in complexity Type I = 0-15% risk Type II = 20-50% risk Type III = approach 100% risk

What are the 3 types of Hawkins classifications for the Talus?

*Type I: Spiral or transverse (Most Common)* Type II: Comminuted Type III: Open

What are the 3 types of femoral shaft fractures?

1. Torus: -cortical buckling on compression 2. Green Stick -Incomplete fracture on tension 3. Plastic Deformity -Bending of the bone without angular break and remodeling

What are the 3 types of incomplete fractures seen in children?

1. Inflammatory (48-hours) 2. Repair (7-14 days) 3. Remodeling (9-24 months)

What are the 3-main phases of fracture healing?

Anterior -40% -Often hyperextension injury Posterior -30% -May be due to direct falls or MVA "dashboard" injury Lateral -20% Rotatory -5% Medial -5%

What are the 5 types of Knee dislocations? Include percentages.

Hamate hook fracture (arrow) -<2-4% -FOOSH Trapezium fracture (arrowheads)

What are the arrowheads pointing at? What is the arrow indicating?

Double-spinous may be noted on APLC view with inferior displacement of SP in Clay-shoveler's fracture

What are the arrows indicating? What type of fracture?

Hill-Sachs deformity/Impaction fracture Bankart lesion (bony or fibrocartilagenous Bankart)

What are the associated compilations with Shoulder dislocations?

1. Scapholunate dissociation 2. Trans-scaphoid perilunate dislocation 3. Ulnar styloid fracture

What are the associated injuries in encountered with a Chauffeur fracture?

PA Caldwell AP Towne B/L laterals PA Water's -May assist in investigation of pathology of the paranasal sinuses

What are the basic radiographic views of the skull?

True

T/F: Pertaining to ACJ injuries, *surgical intervention on a grade III injury should be considered.*

Anterior subcoracoid shoulder dislocation

What *specific* type shoulder dislocation is present?

One fragment displaced >1cm or 45 degrees (13%)

What is a *two-part fracture* according to the Neer classification?

Bedroom fracture with Salter II injury

Where is the fracture? What is the specific type of fracture?

Malunion in the 5th metatarsal fracture

Where is the fracture? What type of disturbed fracture healing is present?

Trimalleolar fracture -Medial oblique and lateral ankle views -Note Bimalleolar fracture and posterior tibial fracture

Identify the type of fracture present.

Malgaigne fracture -Most common unstable type of pelvic fracture -Involves one ipsilateral hemipelvis causing break in anterior and posterior pelvic arches of the pelvic ring. -High incidence of internal organ and vascular injury.

25 year old male after car accident. Apparent shortening of the left lower extremity is note. What is the diagnosis?

Garden Type IV Proximal Femoral Neck Fracture -Complete ->50% displaced

30 year old patient following a major trauma. What is the diagnosis?

Garden Type III Proximal Femoral Neck Fracture -Complete -<50% displaced

70 year old female unable to stand. What is the diagnosis?

Garden Type I Proximal Femoral Neck Fracture -Incomplete -Minimally displaced -Valgus deformity

75 year old female with recent onset of hip pain. What is the diagnosis?

Both Bones (BB) Fracture (Toddlers fracture)

Identify the type of fracture.

Condyle 30% Angle 25% Body 25% Parasymphyseal/Mental 15% Ramus 3% Coronoid process 2%

List the frequency mandibular fractures by location.

Globe-to-wall Bucking

Mechanisms behind "blow out fracture."

*Peripheral* -Well organized mature lamellar bone Intermediate osseous region *-Calcification starts here* Central immature non-ossified focus

A case involving Myositis ossificans (MO), has a *zonal organization phenomenon.* Waht are the zones (3)?

Lateral ankle sprain

A lateral talar process fractures (aka snowboarder fractures) can mimic?

SNAC wrist

A major complication of scaphoid fractures is non-union leading to instability and secondary osteoarthritis and...?

Myositis Ossificans (MO)

A patient presents to the office after blunt trauma to the high during a soccer match. The doctor notices that he has a fever, soft tissue swelling, and poor mobility of the extremity affected. Radiographic features include circumferential calcification with a lucent center, and a *radiolucent cleft string sign* that separates the lesion fro the cortex of the adjacent bone. What is the diagnosis?

1. Anterior Vertebral line 2. Posterior (George's) Vertebral line 3. Spinal Laminar line 4. Posterior Spinous Process line In addition, the evaluation must be completed by careful anatomical survey of the anterior and posterior spinal soft tissues

A reliable way to evaluate alignment of C/S is to loo at 4-lines. What are the four lines?

Myositis Ossificans (MO) Zonal phenomenon involving Deltoid muscle

A young male presents with recent blunt trauma to his shoulder and painful mass. X-ray reveals Heterotopic bone formation. What is the diagnosis?

A. Internally rotated humeral head takes on a rounded appearance B. Vertical dense line in medial humeral head indicating impaction by the posterior glenoid C. Widened glenohumeral joint >6mm, *an important sign*

A. Light Bulb sign B. Trough sign C. Rim sign

Hangman's Fracture Demonstrates traumatically induced injury through both pedicles

Axial CT slice at the level of C2 pedicles indicates what fracture?

Heterotopic ossification (HO)

Bone formation in soft tissues or where bone normally does not exist. The *acquired form* of this is most frequently seen with either *MSK trauma* or spinal cord or CNS injury/paralysis. What is this?

A. Malgaigne fractures B. Open book fracture C. Bucket handle fracture D. Straddle fracture

Brief summary of *unstable* pelvic fractures

Non-union fracture Old trauma

Case: 7 year old trauma. What type of fracture is present?

Torus Fracture (aka "Buckle Fracture') Is an *incomplete pediatric cortical buckle fracture* that occurs during axial loading of a long bone. Most commonly occurs at the distal radius *Mechanism of Injury:* Following a FOOSH injury the force is transmitted from the carpal bones to the distal radius and the point of least resistance leads to fracture of the cortical trabeculae *Location:* Usually the dorsal cortex of the distal radius *Radiographically:* distinct fracture lines are not seen Subtle deformity or buckle of the cortex should be sought *Metaphyseal involvement is typical* In some cases, slight angulation may be the only diagnostic clue *Treatment:* Often self-limiting and do not require surgical intervention, although a manipulation may be required to correct the angulation. Occasionally, a cast may be applied but a splint followed by a period of immobilization will be sufficient

Case: History of acute FOOSH injury. What is the diagnosis?

Bowing fracture (AKA Plastic deformity) Are incomplete fractures of long bones in children (especially the radius and ulna) Often require no intervention with complete remodeling It has been suggested by some that if the angulation is >20 degrees then the fracture needs to be reduced *Mechanisms & Pathology:* -When an angulation longitudinal force is applied to a bone, the bone bends -In Children the bones have a greater degree of elasticity and if the force is low and subsequently released, the bone returns to its normal position -Alternately, if the force is significant the bone may undergo micro-fracturing and plastic deformation and remain in that position -Most Bowing type fractures are seen together with other fractures but may present as an independent phenomenon

Case: History of left FOOSH injury. What is the diagnosis?

*Jefferson Fracture* Aka Burst fracture C1 overhanging C2 -Should be strictly aligned -Greater then 5-mm combined -This indicates potential rupture of the transverse ligament Axial Compression

Case: History of trauma. What type of fracture is present?

Ulna -comminuted (3 bone fragments) -Butterfly fragment

Case: Identify degree of complexity of this fracture, describing its important components

Pathological Fracture Typically occur in an abnormal bone (i.e. fractures through a focal bone destroyed by neoplastic or non-neoplastic processes e.g. infection) Are feared by oncologists because they may lead to immobilization and paralysis in their patients, especially when the spine or lower extremities are affected The presence of pathological fracture may potentially signify systemic and osseous spread of metastasis and portends a poor prognosis

Case: Recent onset of wrist pain. Provide accurate diagnosis of this radiographic abnormali.

80% of *Proximal Humeral Fractures* are minimally displaced *-one-part Neer fracture*

Classification based on four fracture segments:

Weber A -Below sydesmosis *-Stable* -Transverse fracture caused by avulsion

Classify this lateral malleolus fracture.

Weber B -@ the level of the syndesmosis -Without tibial-talar widening -Can be stable or unstable -Oblique type orientation

Classify this lateral malleolus fracture.

Weber C -Above syndesmosis -Unstable -Widening of tibial-talus space due to torn deltoid ligament -Referred by some as *Pott's fracture" that is: 6-cm above the level of the lateral malleolus, and surgical referral and ORIF is required

Classify this lateral malleolus fracture.

2.6-10%

Clavicular fractures are common and account for what percentage of *all fractures?*

The Ottawa Ankle Rule

Clinical diagnosis of suspected ankle fractures and the need for radiographic examination can be successfully determined by applying what rule?

"Dinner fork deformity"

Colles fracture of distal radius

Concept

Colles fracture vs. Smith's fracture

Concept

Concept

All Proximal Femoral Neck Fractures.

Conceptual

Conceptual

Conceptual

Lateral compression AP Compression Vertical Shear

Conceptual Slide

Incomplete pediatric fractures

Conceptual picture.

Image.

Condylar fracture of the distal humerus

Ski-boot or Boot-top fractures When leg is fixed in the ski boot and significant torsion forces are applied during skiing accidents.

Identify the type of fracture.

Calcaneal stress fracture -Seen as vertical parallel line of sclerosis in relation to normal Calcaneal pattern

Identify the type of fracture?

Anterior Tibial-Femoral dislocation

Identify the type of injury present in this image.

Posterior Tibial-Femoral Dislocation

Identify the type of injury present in this image.

Bimalleolar fracture -With associated rupture of deltoid ligament and tib-fib syndesmosis *-Note widening of ankle mortise and instability. -ORIF is required -Unstable fracture

Identify this type of fracture.

Low impact trauma In younger patients high-impact severe trauma, usually a car accident is likely to be involved

Proximal femoral fractures that occur in the elderly is the result of low or high impact trauma?

Older patients Especially who have osteoporosis

Proximal femur fractures tend to occur in older or younger patients?

iliopectineal (iliopubic) Ilioischial teardrop Symphysis pubis Sacrum Acetabulum

Radiographic trauma evaluation of the pelvis requires what lines not to be disrupted?

Subcapital, just below the femoral head (capitis) *Most Common*

Regarding Proximal Femoral Neck Fractures, where is the most common sit for injury?

stronger Less Most

Cortical bone is ________________ on compression and is ___________ resilient to distraction and __________ vulnerable to sheering forces.

False Loss of cervical lordosis *is not* a reliable indication of injury

T/F: Loss of cervical lordosis *is* a reliable indication of injury.

Hangman Fracture *Radiographic features* Bilateral pedicles fracture at C2 Usually associated with anterolisthesis of C2 on C3 Extension of the fracture to the foramina intertransversaria should be evaluated, raising the possibility of vertebral artery injury Extension teardrop C2 or C3 may co-exist CT scanning is required Neurosurgical referral Cord damage only in 25% of cases

Describe important radiographic abnormalities on this image. What is likely diagnosis

Extension teardrop fracture Typically follows forced extension of the neck and avulsion of the anteroinferior corner of the vertebral body Extension teardrop fractures are considered *stable in flexion*, and *unstable in extension* as the ALL is disrupted In older patients the C2 vertebral body is commonly affected due to DDD/DJD and near ankyloses of lower C/S Associated with Anterior Cord Syndrome and Hangman fracture may co-exist Following plain film diagnosis, CT scanning may be required to evaluate for additional trauma

Describe the fracture at C3.

Important

Different types of intercondylar fractures, occur mainly in adults (50%)

Posterior Longitudinal Ligament (PLL) Capsular ligament Interspinous ligaments

Disc space widening and fanning of the spinous processes may strongly indicate disruption of what ligament(s)?

Lunate dislocation (Left) Perilunate dislocation (Right)

Do not confuse lunate dislocations with *perilunate dislocations.*

Pediatric Fractures *Due to vascularity of the periosteum*

Do pediatric or adult fractures heal quicker?

*become entrapped*

During medial epicondyle avulsion (Little League elbow) after extreme valgus force the avulsed fragment may _______________________ in the joint even when there is no dislocation of the elbow.

10% *"Little League Elbow"* -Considered chronic avulsion injury

Epicondylar fractures are common in children and represent __% of all pediatric elbow fractures and usually occur in boys after a FOOSH with avulsion as main mechanism or during strong throwing/pitching.

A. Tubercle B. Distal pole C. Waist D. Proximal pole

Examples of different fracture location in the scaphoid bone

Proximal Femoral Neck Fractures

Extra-articular vs. Intra-articular

A) Valgus Deformity

Fracture of distal tibia and fibula with what type of deformity? Valgus or Varus

Galeazzi fracture-dislocation

Fracture of the distal part of the radius with dislocation of DRUJ and an intact ulna. Name the eponym?

1st-3rd ribs

Fractures of the ______-______ ribs are associated with *high-energy trauma* and potential for serious complications including vascular, neurological and pulmonary damage.

Body -Are further subdivided bases on whether they are traverse, body proper, or fractures involving the talar dome, lateral process, or posterior process. Neck Head

Fractures of the talus can be divided into 3 main regions. What are they?

Type I: -Dorsal distal radial displacement Type II: -Volar distal radial displacement

Galeazzi fractures are classified according to the position of the distal radius. Explain type I and type II.

Used to determine and predict stability and associated complications with required treatment of subcapital neck fractures

Garden Classification of Femoral Neck Fractures (Concept slide)

Symphysis pubis Sacroiliac joints

Greater stability of pelvic fractures depends on whether what joints are intact?

Stress fractures

High-level athletes typically get what type of rib fracture?

Intracapsular repair Intertrochanteric repair Subtrochanteric repair

Hip Fracture Repair

*Extra-articular:* -25-30% -Calcaneal tuberosity avulsion fracture -Calcaneal without talocalcaneonavicular (subtalar) extension *Intra-articular:* -70-75% -Lover's/Don Juan/Casanova fracture of which 10% can be associated with the thoracolumbar fracture *If bilateral Calcaneal fractures are seen, then the spine should be evaluated for fracture as Mechanism of injury is often a large load to the axial skeleton, such as jumping from a second story window.

How can Calcaneal fractures be classified?

Stable and Unstable

How can Pelvic fractures be classified?

*3-column Denis classification system* is being used: *Rules:* -If only 1-column is injured, the fracture can be considered stable. -If *2 or more columns* are injured, the fracture is unstable since significant disruption of spinal ligamentous restraints has occurred.

How can we establish if the fracture is stable or unstable?

*Presence of osteopenia*

How do you differentiate osteogenesis imperfecta from NAI?

Development variant, from incomplete fusion of the sternal ossification centers -5% of population

How does a sternal foramen develop?

Takes twice as long as physiologically expected

How long does a Delayed union take to heal?

No healing for >9 months

How long does a Non-union (pseudoarthrosis) take to heal?

GRUM (Galeazzi-Radius Monteggia-Ulnar)

How to remember Monteggia vs. Galeazzi fracture-dislocations?

Fracture of Hallux sesamoid -Vertically oriented and more irregular vs. bipartite variant.

Identify the fracture present in the image.

Don Juan or Lover's fracture (Intra-articular fracture) -B/L calcaneus Thoracolumbar fracture

Identify the fracture(s).

*Segond Fracture* Suspected ACL tear on this "Tunnel" Inter-condylar view

Identify the fracture.

Tillaux Fracture

Identify the fracture.

Anterior Medial Inferior

In anterior dislocations the humeral head comes to lie ___________, _____________, and ________________ to its normal location and glenoid fossa.

"Shaken baby syndrome"

In child abuse cases, retinal hemorrhage is an indicator of...?

*Holstein-Lewis Fracture*

In some cases of a spiral fracture of the shaft its distal fragment may be displaced in such a way that its proximal end will be displaced radially (i.e. laterally) known by its eponym...?

At the time of impact, the vehicle suddenly accelerates forward. About 100 ms later, the patients trunk and shoulders follow, induced by a similar acceleration of the car seat The patient's head, with no force acting on it, remains static in space. The result is forced extension of the neck, as the shoulders travel anteriorly under the head. With this extension, the inertia of the head is overcome, and the head accelerates forward The neck then acts as a lever to increase forward acceleration of the head, forcing the neck into flexion Radiographic evaluation of *"whiplash injury" is often unrewarding.* CT scanning can help to exclude pillar fracture, MRI can help with soft tissue injury and rarely traumatic disc herniation

Information about Whiplash associated injuries.

Normal Normal height of the lateral tibial condyle appears equal or lower in comparison to medial tibial condyle as seen on this image

Is this image of a normal orientation or abnormal orientation of the tibiofemoral joint?

Extra-articular avulsion fracture of the calcaneus Mechanism involves sudden contraction by the Achilles Tendon

Is this injury extra-articular or intra-articular? What is the mechanism of injury?

Direct blow to sacrum Isolated sacral fractures are usually infrequent (<5%) When sacral fractures are part of complex pelvic injury they are usually vertical and fracture through sacral foramina Sacral fractures associated with pelvic injuries often lead to serious damage to pelvic/abdominal organs and vessels Sacral fractures may also develop as osteoporotic insufficiency or occasionally stress fractures and often are paralleled to the sacroiliac joints When analyzing radiographs for possible sacral fractures one needs to specifically evaluate arcuate liens and sacral foraminae

Isolated sacral fractures are often *transverse* and due to what the of trauma?

C2 pars interarticularis fractures Paravertebral soft tissue swelling Anterior dislocation of the C2 vertebral body

Key radiographic features of Hangman's fracture.

Vascular injury -Popliteal artery *Potentially can lead to limb loss* Knee dislocations are bilateral in 5% of cases Knee dislocations are classified in relation of Tibial displacement compared to Femur

Knee Joint or Femoral-Tibial dislocations are a rare injury, however, it is important to recognize the possible complication that can arise from this injury. What is the complication(s)?

Traumatic break in C2 pars interarticularis C2 body spondylolisthesis C2 extension teardrop fragment

List all (3) of the features of this Hangman's fracture.

Red: Anterior vertebral line Brown: Posterior vertebral line Green: Spinal laminar line Blue: Posterior SP line

List the four colored lines by name.

Fracture of antero-inferior vertebral body (teardrop sign) Loss of anterior height of the vertebral body Focal cervical kyphosis Posterior cervical displacement above the level of injury Fanning of interspinous processes Intervertebral disc space narrowing anteriorly and widening posteriorly Disruption of the spinolaminiar line Anterior dislocation and separation/widening of the facet joints Imaging: Radiography, CT scanning w/o contrast enhancement Further imaging of the spinal cord should be performed to investigate cord injuries (MRI)

List the radiographic features of a Flexion teardrop fracture. *All may not be present in image*

1. Orbit 2. Maxillary sinus 3. Zygomatic bone/arch 4. ?

Name of numbered outlines.

Intra-articular vs. Extra-articular

Note different types of fractures of the 1st MC bone

Key

Note important relationship and positions of the following distal radius fractures

Note significance of talar blood supply to potential neck fractures.

Note significance of talar blood supply to potential neck fractures.

Resulting in mid clavicular Fx commonly angulated.displaced.

Note the mechanisms of muscle actions

69-82% *Reason:* -It is the thinnest part of the bone, and it is not reinforced by attached musculature and ligaments

Of the clavicular fractures, what percentage occurs in the *midshaft?*

Closed Fracture When skin is intact and no communication with the outside air present

Open or Closed fracture?

Open Fracture

Open or Closed fracture?

Posterior rib 8 fracture

Patient presents on radiography a fracture with associated hemopneumothorax evidenced by the visceral pleural line (lung edge) and blunted right costophrenic angle by flattened air-fluid filled inter-pleural space on the right. What is the level and type of rib fracture?

Bankart lesion along with Hill-Sachs deformity

Patient presents with anterior shoulder dislocation and relocation on radiography. What is the diagnosis?

Occult scaphoid fracture *Should require repeat x-ray in 7 days*

Patient presents with history of pain especially in the "snuff box" and FOOSH injury. What is the diagnosis?

14-25 year age range -Kicking and gymnastics are frequently to blame

Pelvic and hip avulsion injuries typically occur in what age range?

AC ligament: sprain CC ligament: intact Joint capsule: intact Deltoid muscle: intact Trapezius muscle: intact Clavicle not elevated with respect to the acromion

Pertaining to Rockwood Classification, what is a type I grade?

AC ligament: Ruptured CC ligament: Sprain Joint capsule: Ruptured Deltoid muscle: Minimally detached Trapezius muscle: Minimally detached Clavicle elevated but not above the superior border of the acromion

Pertaining to Rockwood Classification, what is a type II grade?

AC ligament: Ruptured CC ligament: Ruptured Joint capsule: Ruptured Deltoid muscle: detached Trapezius muscle: detached Clavicle elevated above the superior border of the acromion

Pertaining to Rockwood Classification, what is a type III grade?

*Medial epicondyle is most commonly involved*

Pertaining to epicondylar fractures, which condyle is more commonly involved?

Fibrodysplasia Ossificans Progressive (FOP)

Post-traumatic Myositis ossificans (MO) should be differentiated from a form of severe and progressively fatal congenital disorder known as...?

Non-accidental injury (NAI)

Posterior rib fractures typically indicate...?

Important because of their propensity to damage the small intra-capsular vessels that provide the majority of the blood supply to the femoral head as seen in subcapital, mid and basi-cervical fractures *Extracapsular fractures:* do not usually cause vascular damage The Trochanter is and subtrochanteric fractures are extracapsular injuries *The key feature is to establish whether the fracture is intra- or extra-capsular

Regarding proximal femoral fractures, what is the importance of Intra-capsular fractures?

Rockwood Classification

Rockwood Classification is commonly used for the radiographic-clinical grading of ACJ injury

Salter-Harris Classification of Physeal Fractures

SALTER S-Straight across A-Above L-Lower or BeLow T-Two or Through E R- Erasure of growth or CRush

Type I-V

Salter-Harris Classification.

Non-accidental injuries (NAI)

Skeletal dysplasias and osteogenesis imperfecta can be easily confused with...?

5%

Sternal fractures occur in approx ___________ of blunt chest trauma.

Jefferson Fracture (Burst fracture) is the eponymous name given to a burst fracture of C1 -It was originally described as a four-part fracture with double fractures through the anterior and posterior arches, but three-part and two-part fractures have also been described *Mechanism:* -Diving head first into shallow water -Axial loading along the axis of the cervical spine results in the occipital condyles being driven into the lateral masses of C1 -The Jefferson fracture is not normally associated with neurological deficit although spinal cord injury may occur if there is a retropulsed fragment -Best seen on APOM (radiographically) if >6-mm C1 masses overlap corresponding C2 consider transverse ligament rupture -CT scanning is required -Neurosurgical referral is needed

Summary Slide

Hangman's fracture Extension Teardrop # Hyperextension with superimposed spondylosis

Summary mechanisms of Hangman's Fx & ext. teardrop

Low-energy trauma in osteoporotic bone in the elderly High-energy trauma in young patients *Often intra-articular and frequently comminuted*

Supracondylar and Condylar distal femur fractures usually occur as a result of what?

True

T/F: *CT scanning* will be more helpful and preferred to radiographs due to potentially complex injuries to the chest organs.

True Direct hit to scapula and requires high energy trauma -May result with chest complications as pneumothorax (PTX)

T/F: *Scapula fractures* are uncommon injuries, only account for approximately 3% of all shoulder fractures.

True

T/F: Acute meniscal tears that only involve the *red zone* of the outer fibers can be managed conservatively or heal on their own.

False Head injury *must be* evaluated with Advanced imaging such as CT scanning and MRI Conventional radiography has become practically obsolete

T/F: Conventional radiography for the evaluation of head injury is the gold standard for observing skull fractures.

True

T/F: Fracture callus still remains very vulnerable to shearing forces but may be better stimulate if limited axial forces are applied.

True to injure the medial plateau a large amount of force is required; fractures of the medial plateau are usually seen in conjunction with fractures of the lateral plateau and other bones around the knee joint Mechanism: Driving force of more prominent lateral femoral condyle into the tibial plateau

T/F: Fractures of he lateral plateau are much more common than the medial plateau due to impact by prominent lateral femoral condyle.

True -Most frequently are caused by a crushing injury or axial force such as stubbing a toe *(Bedroom fracture)* *-Toe fractures involving the nail are considered an open fracture and may carry risk of osteomyelitis.*

T/F: Fractures of the toes are the most common lower extremity fractures.

Boxer Fracture (most common type of metacarpal fracture)

The fracture is minimally/mildly impacted, typically transverse fractures of the 4th and/or more typically the 5th metacarpals.

*Stable*

The majority of proximal humeral fractures are stable or unstable?

Gamerkeeper thumb

This avulsion or rupture of the ulnar collateral ligament of the thumb is also known as skier thumb. What is the condition?

Myositis Ossificans (MO) Best by radiography and CT scanning

This condition is a benign process characterized by Heterotopic ossification usually within large muscles. It is essentially metaplasia of the intramuscular connective tissue resulting in extra osseous bone formation.Radiographic-clinical importance stems for its ability to *mimic more aggressive pathological processes (e.g. osteosarcoma). What is the name of this condition?

Bennett fracture-dislocation

This fracture-dislocation of the thumb results from forced abduction of the thumb.

Mallet finger (Baseball finger)

This injury is characterized by inability to extend the finger at the DIP with slight flexion deformity at this joint.

Rolando fracture (aka Comminuted Bennett fracture) -Unstable injury

This is a *three piece or comminuted* intra-articular fracture-dislocation of the base of the thumb.

Extra-osseous or extra-skeletal osteosarcoma Soft tissue osteosarcoma

This should not be confused with Myositis Ossificans (MO). It is important to evaluate if there is a zonal phenomenon present. Note in this x-ray there is evidence of more sclerotic center indicating more aggressive bone forming neoplasm. What is the diagnosis?

Jersey finger (aka Rugby finger or Sweater finger)

This type of injury is where there is avulsion of the *flexor digitorum profundus (FDP) tendon* at the base of the DIP. Most commonly affects the 4th digit as the FDP insertion into the ring finger is anatomically weaker than the middle finger. What is the name of the injury?

*Osteomyelitis.* Open fracture

Toe fractures of the nail are considered (open or closed?) and may carry the risk of ___________?

Mobility Stability

We need to remember that greater ___________ of the cervical spine is sacrificed for its ___________.

*Pathology:* Occurs from a flexion injury of the vertebral body and distraction type injury of the posterior elements The *most common* history is that a back seat passenger restrained by a *lap seatbelt* and involved in a motor vehicle accident or that of a person who has fallen from a height Thoracolumbar junction contributes to 50% of cases *Radiography:* Anterior wedge fracture of the vertebral body with horizontal fracture through posterior elements of distraction of facet joints, disc, and SP *CT scanning is modality of choice* MRI may help with Neurological evaluation following determination of complexity of fracture *Treatment:* surgical or fiberglass plaster with some extension

What are the pathology, radiography, and treatment of a Chance fracture?

Spiral or transverse-oblique fracture are most common and often displaced due to muscle contraction with *"bayonet deformity"*. Can be open or closed

What are the radiographic characteristics of a femoral shaft fracture?

*Transverse fracture in mid patella (most common) "split >60%* Comminuted fracture "stellate" type 25% Vertical fracture 15% *Osteochondral defect usually from medial facet due* to lateral patella dislocation and stress against lateral femoral condyle prominence *Prognosis:* -Guarded in comminuted patella fractures -Surgical referral may be required

What are the radiographic features of a Patella fracture?

Anterior fat pad sign *(sail sign)* -elevated by a joint effusion and appears as lucent triangle on lateral projection Posterior fat pad sign -Not visualized normally Anterior humeral line -Should intersect the middle third of the capitellum in most children, although in children under 4, the anterior humeral line may pass through the anterior third without injury

What are the radiographic features of a supracondylar distal humeral fracture?

Anterior tibial translocation sign *Segond fracture* Arcuate fracture Joint effusion

What are the radiographic signs for an ACL tear?

A. Good fragments apposition and normal bloody supply B. Sufficient immobilization with adequate physiological stress C. Absence of infection or other intervening local factors D. Absence of systemic factors complicating good health and healing

What are the requirements for fracture healing?

Garden Type I and II -Can be treated with surgical fixation -Pins or screws Garden III and IV -Will require total hip arthroplasty

What are the treatments for Proximal Femoral Neck Fractures?

Ankle Mortise and Talus Dorsiflexion & Plantar flexion *only*

What are the true joints of the ankle? What are the only ROM allowed?

*Homolateral:* -With lateral displacement of the 1st to 5th metatarsals, or of 2nd to 5th metatarsals where the 1st MTP joint remains aligned. *Divergent:* -Lateral dislocation of the 2nd to 5th metatarsals with medial dislocation of the 1st metatarsal.

What are the two types of Lisfranc fracture-dislocation?

Fatigue fracture: -Abnormal stresses on normal bone Insufficiency fracture: -Normal stresses on abnormal bone

What are the two types of Stress fractures?

1. Segmental: -2 separate fracture lines producing an isolated segment of bone 2. Butterfly fragment: -Wedge shaped fragment produced at the apex of the maximum force

What are the two types of comminuted fractures?

Carpal arcs of Louis Gilula -Radiographic evaluation of carpal fractures-dislocations

What are these lines called? What are they used for?

Manubrium

What bone in the sternum is more commonly fracture?

Lateral cervical view

What cervical view needs to be cleared first for stability issues before other views are attempted?

Flexed or Extended views

What cervical views should not be performed before ruling out significant spinal injury?

Bayonet Deformity

What deformity is present?

Bayonet deformity May occur when fractured osseous fragments override one another often creating and angular deformity

What deformity is present?

May present significant life threatening complication

What does Intra-cranial hemorrhage indicate?

Healing occurred in abnormally positioned fracture ends

What does Malunion of bone mean?

SP avulsion

What does a "double SP sign" indicate?

Bone is a living tissue composed of different cells. During injuries, cell damage creates intra-osseous edema. Best detected on MRI. Bone bruise may accompany other injuries such as ligaments sprains/tear of the ankles and knees

What does a Bone Bruise consist of?

Absent callus often with synovial fluid or infected exudate intervening between fracture ends.

What does atrophic disruption of bone healing mean?

Abnormal exuberant callus

What does hypertrophic disruption of bone healing mean?

Week callus with insufficient vascularization and new bone formation

What does hypotrophic disruption of bone healing mean?

Subchondral lucency/osteopenia line of the talar dome that occurs secondary to subchondral atrophy 6-8 weeks after a talar neck fracture. This indicates that there is sufficient vascularity in the talus, and is therefore unlikely to develop avascular necrosis since the blood flow and bone resorption were re-established.

What does the Hawkins sign describe?

Tripod Fracture *2nd m/c facial fracture after nasal fracture (40% of mid face fracture)* M/c mechanism is a blow to malar region of the face Complications: -Trismus -Trapped temporalis msucle -loss of sensory distribution along maxillary (V2) nerve distribution *Best Imaging modality* CT scan

What facial fracture is present?

Flexion teardrop fracture *Description:* Capsular, disc and posterior ligaments disruption and anterior compression fracture of the vertebral body which results from a severe flexion loading injury *Mechanism:* Hyperflexion and compression (e.g. MVA, diving into shallow water etc.) *Radiographic features:* (Best seen on lateral view) -Prevertebral swelling associated with anterior longitudinal ligament tear -Teardrop fragment that is typically wider in the horizontal diameter usually as a result of anterior vertebral body fracture (may be superior, inferior or multiple fragments) -Posterior vertebral body migration/displacement into the spinal canal -Spinal cord compression from posterior vertebral body displacement (anterior cord syndrome) -Fracture of the spinous process (may or may not be present) -Very poor prognosis (80% are paralyzed at the time of the accident)

What fracture is present?

Hangman's fracture

What fracture is present?

Malgaigne fracture on the right

What fracture is present?

Straddle fracture

What fracture is present?

Straddle pelvic fracture Unstable and quite frequent Superior pubic rami and Ischiopubic junction bilaterally Often serious bladder and urethral injury occurs In males: -Severe urethral injuries may result in stricture

What fracture is present?

Jones Fracture (Dance Fracture) *-Distal horizontal fracture line without intra-articular extension.*

What fracture is present? What is the key diagnostic and DDX feature?

Monteggia fracture-dislocations

What fracture-Dislocations comprises a fracture of the ulna shaft and dislocation of the proximal radio-ulnar joint (PRUJ)

Chauffeur fracture (aka *Hutchinson fractures* or *backfire fractures*)

What fractures are intra-articular fractures of the radial styloid process?

Type II Odontoid Process Fracture

What grade and type of fracture is present?

Type II Odontoid Process fracture May be undetected on plain film As discussed CT scanning w/o contrast is required to fully evaluate cervical spine trauma

What grade and type of fracture is present?

Type II Odontoid Process Fracture Note other significant changes related to DISH that may have contributed to this fracture (rigid spine)

What grade and type of fracture is this?

LeFort I

What grade of LeFort fracture is present?

LeFort I & II

What grade(s) of LeFort fracture is present?

Hematoma and inflammatory mediators within first 48-hours initiate chemotaxis with phagocytes and repair cells being drawn to fracture site (shortest phase)

What happens during the Inflammatory phase?

When population of cells will sufficiently evolve into fibroblasts, chondrocytes and osteoblasts the osteoid and bone mineralization will continue. Remodeling phase on average may take 9-24 months.

What happens during the remodeling phase?

Cells involved during initial inflammation will gradually begin to form granulation tissue and remove unwanted material and damaged cells. Curing this phase within 7-14 days hematoma becomes vascularized and may appear more translucent on X-rays

What happens during the repair phase?

Zonal phenomenon *MO-is a "don't touch or leave it alone lesions"*

What helps to differentiate Myositis ossificans from more aggressive lesions (e.g. extra-osseous osteosarcoma, soft tissue sarcomas)?

Elbow apophyseal ossification center CRITOE (1-3-5-7-9-11 years)

What is *CRITOE*?

Angle between two tangent lines drawn across the anterior and posterior borders of calcaneus on the lateral view. If Bohler's angle is less than 20-degrees it indicates a Calcaneal fracture

What is Bohler's Angle?

That is a slippage of the torn end of the ulnar collateral ligament superficial to the Adductor Pollicis muscle causing interposition of the Adductor Pollicis msucle between the ulnar collateral ligament of the MCP joint. This prevents healing and is an indication for surgical repair.

What is a *Stener lesion*?

No fragments displaced (80%)

What is a *one-part fracture* according to the Neer classification?

Displacement of the surgical neck and either the greater or lesser tuberosity (3%)

What is a *three-part fracture* according to the Neer classification?

Common in anterior shoulder dislocation -Related to detachment of the anterior inferior labrum from the underlying glenoid as a result of anteriorly dislocated humeral head impacting against the labrum or following the avulsion by the inferior glenohumeral capsular ligaments. -It may be labral only ("*Cartilagenous or soft Bankart*") or "*Bony bankert*" with detached fragment of inferior glenoid rim.

What is a Bankart lesion?

Fractures of the distal radius. It is also sometimes termed the *dorsal type Barton fracture* in an attempt to distinguish it from the volar type or reverse Barton fracture. Barton fractures extend through the dorsal aspect to the articular surface but not to the volar aspect. Therefore, it is similar to a *Colles fracture* but with *intra-articular communication* *-Pronator Fat Pad* location and displacement suggestive of *intra-articular effusion*

What is a Barton fracture?

Lateral compression force (car accidents) Vertically-orientated fractures through the superior and inferior pubic rami on one side and *contralateral* sacro-iliac joint disruption Unstable injury potentially resulting in serious complications involving internal organs and vascular structures

What is a Bucket handle fracture?

Are a type of comminuted compression fracture which results in disruption of the posterior vertebral body cortex with retropulsion of fragments into the spinal canal When it involves the thoracolumbar level, it tens to occur *between T9 and L5 level* *Pathology:* burst fracture, is a result of high energy axial loading Fall and landing on the feet, from a height and/or MVA The nucleus is driven into the vertebral body below causing "burst" All patients require a CT to assess this injury and evaluate the extent of retropulsed fragments which may enter spinal canal or in complex cases retropulse through spinal canal even into posterior back

What is a Burst fracture?

Typically, corner fracture that is chipped rather than avulsed.

What is a Chip fracture?

Is a fracture/dislocation of the mid-tarsal joint (Chopart joint) of the foot -Talonavicular and calcaneocuboid joints -Will separate the hindfoot from the mid foot *-Commonly fractured bones are the calcaneus cuboid and navicular* -The foot is usually dislocated *medially and superiorly* as it is plantar flexed and inverted -Result of *high energy impact* (fall from height or road traffic collision) -When the foot is exerted, lateral displacement occurs.

What is a Chopart injury?

Moor fracture

What is a Colles fracture called with it is associated with a dislocation of the ulna?

Stable iliac wing fracture -Resulting from a direct blow Treatment -conservatively but occasionally may be comminuted and may deserve surgery

What is a Duverney fracture?

Bilateral cervical facet dislocation is a flexion distraction type of injury Results from hyperflexion and traction and also reported buckling force to be involved If flexion/distraction injury occurs whilst cervical rotation is present-a unilateral facet dislocation may occur Flexion/distraction force may tear capsular ligaments leading to facet of the above vertebra overriding or perched on the one below 50% anterolisthesis of vertebral body may be seen radiographically with perched and dislocated facets Unstable injury since more than a single column involvement has occurred CT & MRI may help to evaluate further osseous, articular or neurological abnormalities

What is a Facet dislocation?

Traumatic spondylolisthesis of C2 Most cases related to MVA Due to hyperextension and traction of the upper cervical spine leading to b/l brea of pars interarticularis of C2 and disruption of the discovertebral junction *Shares some similarity with injury induced by judicial execution during hanging by the neck* Considered *unstable* If fracture line extends to the foramina intertransversaria, vertebral artery damage may occur

What is a Hangman's fracture?

Posterolateral humeral head impaction fracture as the humeral head pressed against the antero-inferior part of the glenoid. -Often associated with Bankart lesion of the glenoid -Classically: a sclerotic vertical line extending from the top of the humeral head towards the shaft -Occasionally termed a *"hatchet deformity"*

What is a Hill-Sachs lesion?

Normal stresses (i.e. normal weight bearing, walking) applied to osteoporotic (involuted/ insufficient) bone. For example, vertebral osteoporotic, sacral insufficiency fractures.

What is a Insufficiency Fracture due to?

Burst fracture of C1 (Axial loading) C1- represents an osseous ring that ossified between 3-6 years old age. Thus a ring structure may fracture in several locations *Mechanism:* "diving head first" compression of occipital condyles into lateral masses of C1 *Clinically:* No cord damage may be seen due to separation of fragments, potentially moving apart w/o direct cord damage by the osseous fragments

What is a Jefferson fracture?

Is a combination of oblique or spiral fracture of the proximal fibula and unstable ankle injury. *High ankle sprain* Manifests by widening of the ankle joint due to distal tibiofibular syndesmosis and/or deltoid ligament disruption and sometimes fracture of the medial malleolus

What is a Maisonneuve fracture?

Unstable Vertical shear force Ipsilateral hemipelvic fracture -Pubic rami -Sacroiliac joint or sacral ala *Clinically:* -Short limb on the side of fracture -Other major complications -AP pelvis view will show Malgaigne fracture quite readily *Treatment:* -Surgical -Prognosis may be poor if internal injury is present

What is a Malgaigne fracture?

Is a vertebral facet joint whose inferior articular process appears to sit (perched) on the ipsilateral superior articular process of the vertebra below. Any further anterior subluxation will result in dislocation, with one facet "jumping" over the other and becoming locked in this position leading to "Jumping facets" that will led to overriding and "locked facets" frequently seen in complete bilateral facet dislocation

What is a Perched facet joint?

Results from an abduction-external rotation mechanism when anterior tibiofibular ligament avulses the anterolateral corner of the distal tibial epiphysis *Considered a Salter-Harris 3 injury of distal tibia* *Key feature:* -Anterolateral distal tibial epiphysis avulsion because on the medial aspect of growth plate it is fused ahead of the lateral physis fusion.

What is a Tillaux fracture?

Undisplaced spiral fractures usually of the tibia in toddlers either due to fall or when the leg is stuck in between the wooden spacing of the crib/cot. Aka: *B*oth *B*ones fracture

What is a Toddler fracture?

May involve medial tubercle (aka *Cedell fracture*) or lateral tubercle (aka Shepherd fracture*) of the posterior process of talus. -Medial tubercle fracture is uncommon and is caused by forced dorsiflexion and pronation. The posterior talofibular ligament attaches on the lateral tubercle and flexor Hallucis longus runs between these tubercles. Infused ossification center of the lateral tubercle forms the os trigonum. *Mechanims of injury:* -Caused by inversion *Treatment:* -Short leg cast or ankle brace is used until signs of union appear. -If there is non-union, then excision of the fragment is undertaken.

What is involved in a posterior talar process fracture? Mechanism of injury? Treatment?

Hill-Sachs deformity/Impaction following anterior-inferior shoulder dislocation

What is present on this CT scan?

Bone (Intra-osseous edema) bruise Low signal intensity in the distal pole of the scaphoid bone

What is present on this T1 coronal wrist MRI?

Persistent Ossiculum terminale Not to be confused with Type I odontoid fracture

What is present on this X-ray?

Os odontoiedeum Mimics Type II Odontoid Process Fracture

What is present on this X-ray? *Hint:* not what you think

Osseous Bankart lesion

What is present on this coronal reconstructed CT slice?

Hill-Sachs deformity and very subtle bony Bankart lesion.

What is present on this radiograph?

Os Odontoiedeum (Hypoplastic type) Mimics a Type II Odontoid Process fracture

What is present on this x-ray?

*9-12 years of age* FOOSH

What is the *peak incidence* of a Galeazzi fracture-dislocation? Common mechanism of Fx?

Scaphoid fracture through the distal pole

What is the *specific* diagnosis?

Scaphoid fracture through the waist

What is the *specific* diagnosis?

Type III Supracondylar distal humeral fracture *Type I: un-displaced* *Type II: displaced with intact cortex* *Type III: Complete displacement (usually posterior)*

What is the *specific* diagnosis?

Divergent Lisfranc injury

What is the *specific* type of fracture present?

Homolateral Lisfrance injury with avulsion fragment

What is the *specific* type of fracture present?

Triquetral fracture -Note important *Dorsal radiocarpal ligament attachment*

What is the 2nd most common carpal bone fracture, after the scaphoid?

Normal ACL angle should be aligned along femoral condyles

What is the Blumensaat's line?

FBI sign or fat-blood-interphase: *Sign of lipohemarthrosis* Lipohemarthrosis that results from an intra-Articular fracture with escape of fat and blood from the bone marrow into the knee joint Most frequently associated with a tibial plateau fracture or distal femoral fracture Patella fractures may also result in lipohemarthrosis

What is the FBI sign and what fracture does is pertain to?

Proximal Humeral Fracture Four-part Neer fracture

What is the Neer classification present?

(Aka Danis-Weber) Is a popular classification of lateral malleolar fractures, relating to the level of the fracture and its relation to the ankle joint and tib-fib syndesmosis. Predicts the degree of ankle stability and considers approach to treatment. Weber A: -Below syndesmosis -Stable Weber B: -@ the level of syndesmosis -Syndesmosis may remain intact. -Sometimes deltoid ligament may be torn and tib-fib widening Weber C: -Above Syndesmosis -Unstable with disruption of syndesmosis and fracture of the medial malleolus -Deltoid ligament may be torn

What is the Weber ankle fracture classification?

Calcaneal stress fracture on MRI

What is the arrow indicating?

Abnormal pronator sign (arrow) Compared to a normal pronator fat pad (right)

What is the arrow indicating? Right side indicates a normal ____________?

*Arcuate Sign* Indicates avulsion of fibular attachment of biceps femoris and lateral collateral ligament -High suspicion of *ACL tear* *-Potential posteriolateral instability and failed ACL reconstruction if this injury was undetected earlier*

What is the arrow pointing at? What type of injury does this indicate?

Tibial spine avulsion (black arrow) -May occur especially in young or adolescent patients as a result of a pull and *tear by the ACL* Segond Fracture (white arrow) -Is an avulsion likely to be caused by the pull of lateral capsular knee ligaments and possibly an IT band and has a high association with ACL tears

What is the black arrow indicating? What is the white arrow indicating?

Duverney Fracture

What is the blue arrow indicating?

Overuse injury -Seen in athletes (esp. running and aerobics, and military recruits)

What is the cause of a Calcaneal stress fracture?

Metaphyseal corner fracture Induced by twisting, pulling and forceful shearing injuries

What is the classic presentation that indicates NAI?

AVN of scaphoid, evidenced by increased sclerosis fragmentation and cytic changes

What is the complication of a waist fracture in the scaphoid?

Scapholunate Advanced Collapse (SLAC)

What is the condition called where a pattern of wrist malalignment that has been attributed to post-traumatic or spontaneous osteoarthritis of the wrist and can develop following scaphoid fractures and non-union or other arthritic conditions. In addition, this may lead to DISI or VISI (dorsal intercalate segmental instability vs. volar intercalate segmental instability). Dorsal or volar lunate tilt is crucial in evaluating DISI vs. VISI.

Growing skull fracture or Leptomeningeal cyst Develops prior to closure of skull (<3 y.o) as a result of tear in the dura followed by leptomeningeal and brain herniation with CSF pulsation. Not a true cyst.

What is the condition?

Separation of fibrous joints or fibrocartilagenous joints often seen as suture diastasis in the scull and symphysis pubis

What is the definition of Diastasis?

Burst fracture *Radiographic features:* burst of vertebral body fragments well demonstrated on axial CT Loss of posterior vertebral height on lateral radiographic views *retopulsed fragments in the spinal canal* Interpedicular distance widening due to "burst" of body *Clincally:* stable or unstable, needs Ct and MRI if neurological damage is established. Neurosurgical consult may be needed

What type of fracture is present?

Burst fracture Comminuted fracture often with posterior displacement (retropulsion) of fragments and potential cord damage

What type of fracture is present?

Cervical wedge fracture at C6, uncomplicated injury involving a single column-stable fracture. Note some anterior wedging of the C6 body *Compression "Wedge compression fracture:* cervical, thoracic, and lumbar Hyperflexion injuries to the vertebral body resulting from axial loading Most commonly affecting the anterior body aspect Wedge fractures are considered a *single-column (i.e. stable) fracture* May occur in the C/S but *typically seen in the Thoracic and T/L region Radiography: No posterior body retropulsion, wedging of the anterior body typically of superior end-plate impaction *Some osteoporotic thoracic compression fractures may share great resemblance* Clincally: Stable and *no neurological deficit*

What type of fracture is present?

Chance (lap seat belt injury) fracture

What type of fracture is present?

Chance fracture or Seat belt fracture Note horizontal fracture through the body and posterior elements. AP view may show widening of interpedicular distance

What type of fracture is present?

Clay-shoveler fracture

What type of fracture is present?

Clay-shoveler fracture Note obliquely oriented SP fracture of SP of C7 It is important to remember that it is a *stable injury* and may only be discovered some time (years) later for unrelated radiographic investigation

What type of fracture is present?

Depressed Skull fracture (thick arrow) Pneumocephalus and comminuted fragmetns pushed inwards into the brain parenchyma and often tearing the overlying meninges

What type of fracture is present?

Duverney fracture

What type of fracture is present?

Femoral condyle fracture May occur as "T" or "Y" configuration and always intra-articular Clinically can be complicated by delayed healing and comminuted fragments Tibial plateau fracture may co-exist and should be specifically sought

What type of fracture is present?

Flexion teardrop fracture *Description:* Capsular, disc and posterior ligaments disruption and anterior compression fracture of the vertebral body which results from a severe flexion loading injury *Mechanism:* Hyperflexion and compression (e.g. MVA, diving into shallow water etc.) *Radiographic features:* (Best seen on lateral view) -Prevertebral swelling associated with anterior longitudinal ligament tear -Teardrop fragment that is typically wider in the horizontal diameter usually as a result of anterior vertebral body fracture (may be superior, inferior or multiple fragments) -Posterior vertebral body migration/displacement into the spinal canal -Spinal cord compression from posterior vertebral body displacement (anterior cord syndrome) -Fracture of the spinous process (may or may not be present) -Very poor prognosis (80% are paralyzed at the time of the accident)

What type of fracture is present?

Flexion teardrop fracture One of the most severe injury of the cervical spine *Causing:* -Anterior cervical cord syndrome -Quadriplegia *Pathological Mechanisms: -Typically occurs from severe flexion and compression, most commonly at C5-C6 -Will disrupt annular disc fibers, fracture vertebral body and tear posterior spinal/capsular ligaments *Clincally:* -Anterior cord damage may develop due to posterior displacement of vertebral fragments

What type of fracture is present?

Greenstick fracture Incomplete pediatric fracture

What type of fracture is present?

Greenstick fracture Potential complications: -Ischemia

What type of fracture is present?

Hangman Fracture (aka traumatic spondylolisthesis of C2) Is a fracture which involves the pars interarticularis or pedicles of C2 on both sides, and is a *result of hyperextension and distraction injury* Hyperextension, such as high speed MVA, is in fact the *most common* association

What type of fracture is present?

Hangman fracture (aka traumatic spondylolisthesis of C2

What type of fracture is present?

Hangman's Fracture Note pedicular C2 fracture and anterolisthesis of C2 body

What type of fracture is present?

Hawkins Type I undisplaced talar neck fracture

What type of fracture is present?

Hawkins Type I undisplaced talar neck fracture -Note: Subtle cortical off-set at the junction of talar head-body region

What type of fracture is present?

Intertrochanteric proximal femoral fracture *Extra-capsular Proximal femoral fracture* Often the result of violence and related to falls or car or pedestrian vs. car accidents Intertrochanteric and subtrochanteric fractures more readily develop in younger patients because of more significant forces involved such fractures are often comminuted Complications related to vascular damage or fat pulmonary embolisms may occur

What type of fracture is present?

Jefferson Fracture

What type of fracture is present?

Jones Fracture (Dance Fracture)

What type of fracture is present?

Leptomeningeal Cyst (growing fracture) Most found in pediatric skull <3y.o 1% of skull fractures Develops due to meningeal tear and interposition of Leptomeningeal and CSF with or w/o cerebral parenchyma

What type of fracture is present?

Hawkins Type II -Displaced fracture with subluxation or dislocation of the subtalar joint and a normal ankle joint. -20-50% risk of AVN

What type of fracture is this?

Hawkins Type III -Displaced fracture with body of talus dislocated from both subtalar and ankle joint -Approach 100% risk of AVN

What type of fracture is this?

Mandibular fracture Common especially among young men Traditionally the mandible and base of skull are though to form a complete bony ring, linked to skull by TMJ Theoretically this should mean that the mandible should fracture in more than one place, similar to the pelvis However, more than 40% of mandibular fractures are *unifocal* Mandible fractures are a common injury because of the mandible's prominence and relative lack of support As with any facial injury, consideration must be given to secure the airway or to obtain hemostasis Complications may include, airway obstruction, aspiration of broken teeth and infection including other major and minor complications

What type of fracture is present?

Non-union fracture of distal tibia and fibula Pseudoarthrosis No good callus formation

What type of fracture is present?

Orbital blow-out fracture

What type of fracture is present?

Pseudopathological fracture Some complete displaced neck fractures may mimic pathological fractures Displaced femoral neck fractures require surgical fixation *Clinically:* During complete displaced neck fractures the patient will be unable to walk or stand and their limb may appear shortened and externally rotated

What type of fracture is present?

Rolando or Comminuted Bennett Fracture

What type of fracture is present?

Salter-Harris Type II

What type of fracture is present?

Salter-Harris Type II Good prognosis *Thurston-Holland fragment* -Triangular bone fragment

What type of fracture is present?

Salter-Harris Type IV

What type of fracture is present?

Segond fracture-avulsion Important radiographic clue to ACL tear

What type of fracture is present?

Snowboarder fracture

What type of fracture is present?

Split fracture of tibial plateau Noted that in younger patients significant axial loading forces are likely to be involved Injuries to knee ligaments may develop in 10-15% cases in association with this injury

What type of fracture is present?

Sprung-Pelvis Fracture-dislocation (aka *"open-book" pelvis*) Due to forced separation of the posterior ring through one or both sacroiliac joints with disruption of the pubis symphysis and anterior ring diastasis. Severe injury CT scan is needed to fully evaluate the extent of this injury and surgical care Radiographically: -Iliac wings appears asymmetric

What type of fracture is present?

Subcapital neck fracture?

What type of fracture is present?

Subtrochanteric fracture *Extra-capsular proximal femoral fracture* Often 2-inches inferior to lesser trochanter and generally are infrequent Their origin and clinical manifestations are similar to intertrochanteric fracture Surgical fixation (ORIF) is required

What type of fracture is present?

Tibial tuberosity avulsion

What type of fracture is present?

Tillaux fracture OR Anterolateral distal tibial avulsion with not fully closed physis

What type of fracture is present?

Toddlers Fracture -Spiral fracture

What type of fracture is present?

Trampoline transverse fracture through proximal tibial metaphysis Growth plate appears intact

What type of fracture is present?

Un-united Scaphoid (carpal navicular) fracture through proximal pole *SNAC Wrist* Scaphoid Non-union Advanced Collapse

What type of fracture is present?

Unilateral facet dislocation -Due to flexion and rotation injury -In unilateral facet dislocation, anterior body displacement is <50%, also sudden facet rotation and alignment loss is present -Overriding facets may typically produce *"bow tie" appearance due to sudden facet rotation on lateral cervical view* -This may be stable or potentially unstable injury

What type of fracture is present?

Extension teardrop fracture of C2

What type of fracture is present? Hint: C2

Banana-type fracture Complete bisphosphanates-induced fracture that resembles insufficiency type fracture

What type of fracture is present? What is this a direct result from?

Unilateral facet dislocation Radiographically ,*"bow tie"* facet appearance -On lateral view with sudden discordant facet alignment -On APLC view SP are not aligned indicating rotatory deformity of the involved segment

What type of fracture is present? What radiographic feature is present?

Lateral tibial plateau fracture FBI sign In older patients the fracture may be undisplaced Impaction of lateral tibial plateau and difficult to visualized. Thus FBI sign serves as an important confirmation of intra-articular fracture

What type of fracture is present? What sign is present?

Wedge fracture with end-plate impaction and *"Step sign"* Considered stable

What type of fracture is present? What sign is the white arrow indicating?

Cervical articular pillar fracture commonly at C4-C7 with C6 (40%) cases Can be frequently missed on radiographs CT imaging with axial and reconstructed slices can be helpful Considered stable injury

What type of fracture is present? What vertebra are involved? Is this considered a stable or unstable injury?

Mechanisms of Chance fracture or seat belt fracture

What type of fracture is the drawing depicting?

Osteochondral injury of talar dome

What type of fracture is the red arrow pointing to?

Subcapital neck fracture

What type of fracture is the red arrowhead indicating?

*Simple wedge fracture* More common in the thoracolumbar region and considered stable. Note anterior body wedging at L2. Likely due to flexion/compression

What type of fracture is the white arrow pointing at?

Flail rib

What type of fracture is this conceptual image demonstrating?

Hawkins Type I -Non displaced fracture -0-15% risk of AVN

What type of fracture is this?

Basilar skull fracture Specifically: Basilar temporal fracture Often better identified clinically than Radiographically *Radiographic signs of basilar fracture can be very subtle or not seen at all* M/C area: Sphenoid and temporal bones -Greater than 75% of cases with complete or partial obscurantion of the Sphenoid sinus, best detected on lateral skull projections Tearing of the meninges is common *Clincally:* CSF rhinorrhea and/or otorrhea, *Battle sign and Raccoon eyes sign can be observed* *Best Modality:* CT scanning Neurosurgical referral is imperative

What type of skull fracture is present?

Depressed Skull Fracture

What type of skull fracture is present?

Linear skull fracture 80% of skull fractures M/C found in the parietal and temporal bones Identified as radiolucent line that unlike vascular grooves *tends to cross sutures*

What type of skull fracture is present?

Ping Pong fracture Aka Pond skull fracture Variation of depressed skull fracture (incomplete) that may be seen in new born infants as an inward buckling of the calvaria without osseous fragments *Pathology:* -Skull bones of a new born infant may fracture without osseous and periosteal break, similar to "green stick" fracture -May develop during labour or forceps delivery or postnataly after direct blunt trauma (dropped infant) *Radiographically:* -Continuous smooth inward indentation of the skull calvaria

What type of skull fracture is present?

Depressed skull fracture *Considered an open fracture* (15% of all skull fractures) *Best modality for Dx: CT scanning* If depressed fragments extend >0.5-1.0cm, neurological referral is crucial Depressed fractures are associated with serious complications and high mortality: -Epidural, subdural, and subarachnoid hemorrhage -Meningitis -Pneumocephalus -CSF leak

What type of skull fracture?

Sutural Diastasis Specifically: Traumatic lambdoid sutural diastasis May be traumatic or non-traumatic due to pathology or rarely physiological changes related to rapid growth spurts *Traumatic Sutural Separation* (diastatic fracture) may be seen with or without fracture *Dx:* at birth >1cm, age 2 >3mm, age 3 >2mm *Other Pathology that may separate normal sutures:* -Raised ICP/Hydrocephalus -Neuroblastoma -Leukemic and lymphoma cells infiltration

What type of skull fracture?

AP Towne's

What type of skull view is present?

Lateral Skull

What type of skull view is present?

PA Caldwell

What type of skull view is presented?

Oblique fracture Bayonet deformity

What type of specific type of femoral shaft fracture is present? What deformity is also present?

Intracapsular repair ORIF treatment of subcapital fractures

What type of surgical repair is present?

Repair of intertrochanteric fracture with ORIF

What type of surgical repair is present?

Blunt trauma -MVA -Falls -Assaults

What type of trauma is the most common injury occurring in 50% of cases?

Y-scapula view Anterior GH dislocation -Subcoracoid Position

What type of view is this? What is the diagnosis?

Calcaneal axial (Harris) view Comminuted intra-articular Calcaneal fracture

What type of view is this? What part of the body is being imaged? Is this intra or extra-articular?

Hangman's fracture and extension teardrop avulsion fracture at C2 Traumatically induced pars interarticularis break

What type(s) of fracture is present?

Right mandibular body and left condyle fracture

What type(s) of mandibular fracture(s) is/are present/

Lateral patella dislocation and osteochondral defect Sunrise view *(Most helpful)*

What view is this? What is being demonstrated in the image?

*"Aviator's asatragalus fractures or Aviator's fractures"* -Because earlier pilots were crashing their airplanes and suffered characteristic fractures in which their foot was dorsiflexed on the floorboard of the plane.

What were Talar neck and to some degree talar body fractures once known as?

Occurs at the base of the 5th MT

Where does a Jones fracture (Dance Fx) occur?

Displaced directly below, and a little medial to the glenoid -High complication rate of secondary vascular, neurological and soft tissue injury

Where does the humeral head lie after an Inferior Shoulder Dislocation?

Ischial tuberosity Associated muscle -Hamstring muscles

Where is the avulsion? What is the associated muscle?

Left AIIS avulsion Associated muscle: -Rectus Femoris muscle

Where is the avulsion? What is the associated muscle?

Right AIIS avulsion Associated muscle -Rectus Femoris muscle

Where is the avulsion? What is the associated muscle?

Right ASIS avulsion Associated muscle -Sartorius muscle Part of Iliac Wing avulsion Associated muscle -Abdominal muscles

Where is the avulsion? What is the associated muscle?

Both lateral femoral and tibial condyles Also ACL is torn along with medial meniscus

Where is the bone bruise (intra-osseous edema) in this MRI?

Anatomical snuffbox

Where is the clinically classical pain present in a scaphoid fracture?

Young people with high-energy should trauma (falls, FOOSH in sports etc.)

Who typically present with Anterior Shoulder dislocation?

Osteogenesis imperfecta DDx: NAI In Children

Young child presents on radiography a slender, transverse fracture of the femoral shaft in addition to *bowing deformity* of both femurs. What is the diagnosis?


Kaugnay na mga set ng pag-aaral

Genetics Chapters 13,14,16,17, application C, DSM 13

View Set

Solving Quadratic Equations: Completing the Square (Continued) Assignment

View Set

Cybersecurity Fundamentals-Definitions

View Set

Administration Theory and Behavior

View Set

Religious pluralism and theology

View Set

What 'groupthink' is, an example and strategies teams can use to avoid this

View Set

Business Strategy Lecture 5 - Capabilities

View Set